LSAT and Law School Admissions Forum

Get expert LSAT preparation and law school admissions advice from PowerScore Test Preparation.

 Administrator
PowerScore Staff
  • PowerScore Staff
  • Posts: 8917
  • Joined: Feb 02, 2011
|
#36508
Complete Question Explanation

Flaw in the Reasoning. The correct answer choice is (D)

This stimulus introduces a letter to the editor complaining of the reasoning in a recent article on
speed limits. In that article, it was noted that areas with lower speed limits had lower vehicle fatality
rates. But the letter writer concludes that it will not be that way for long, based on the fact that
vehicle-related fatalities are increasing in areas with lower speed limits.

The question stem asks why the reasoning in the letter writer’s argument is flawed. Whenever we
see simple numbers comparisons, we should be wary of the author’s tendency to draw unwarranted
conclusions. The problem here is that a simple increase in the number of vehicle related fatalities
does not provide sufficient evidence to logically draw any conclusions about whether these fatalities
are attributable to the lower speed limits. If we are seeking to determine whether or not safety is
increased by lower speed limits, a more relevant comparison would be between the respective
fatalities of high vs. low speed limit areas.

Answer choice (A): Reliance upon empirical evidence cited in the original article is not a fl aw in
the letter writer’s argument—it is quite common on the LSAT to see two different viewpoints or
interpretations based on the exact same evidence. The author of the letter is not refuting the evidence
provided by the original report, but rather the interpretation of that evidence, so this answer choice is
incorrect.

Answer choice (B): The term “often” is extremely vague, and provides no insight into the relative
likelihood of fatalities at high speeds vs. low speeds. The reason the conclusion in the stimulus
is fl awed is that it rests on a shaky premise, not that it fails to consider all outside evidence. This
answer choice does not provide an effective attack on the stimulus’ reasoning.

Answer choice (C): The fact that some drivers don’t want to drive any faster plays no role in the
editorialist’s argument, since an increased speed limit would not require anyone to drive faster. The
fact that some don’t wish to drive faster is irrelevant, and certainly does not represent a fl aw in the
author’s reasoning, so this answer choice should be eliminated.

Answer choice (D): This is the correct answer choice. If vehicle fatality rates are increasing
everywhere, not just in the low speed limit areas, then we cannot logically draw any justifiable
conclusions about the increase in fatality rates that has taken place in the low speed limit areas, and
raising the speed limit based on these figures would not necessarily be advisable.

Answer choice (E): The letter writer does provide some evidence (though questionable) against the
opposing viewpoint—the evidence that the vehicle fatality rate is increasing in the low speed limit
areas. This evidence may be weak, but the claim is presented, so this answer choice is inaccurate and
incorrect.
 ege222
  • Posts: 7
  • Joined: May 02, 2014
|
#14618
I completely ruled out what turned out to be the correct answer of D. I chose E. Why is D correct and why isn't E?.

Thanks in advance,
Elizabeth
 Lucas Moreau
PowerScore Staff
  • PowerScore Staff
  • Posts: 216
  • Joined: Dec 13, 2012
|
#14620
Hello, Elizabeth,

E is tricky. The use of the word "evidence" doesn't mean the author has to produce hard facts, but a claim such as is presented - that vehicle fatality rates are rising in areas with lower speed limits - should be taken as true in this case, and as evidence. So the speaker is presenting a claim as evidence for the opposing viewpoint, so E is incorrect.

As far as D goes, D would mean that the author's premise does not lead to his conclusion. If vehicle fatality rates are rising in areas with higher speed limits as well as lower, that undercuts the author's argument that rising fatality rates in lower speed limit areas show that lower speed limits are causing the problem. D is the correct answer choice, because the author should have considered those possibilities in making his argument.

Hope that helps,
Lucas Moreau
 ltowns1
  • Posts: 61
  • Joined: May 16, 2017
|
#47787
Hi, I have a question . First, I didn’t like B or D to be honest. I thought (D) was a tempting incorrect answer. I chose (B), but I see why it is incorrect. One thing that confuses me about (D) though is exactly the part that makes it right. When they say “other areas are also rising” how are we to conclude those areas are places with high speeds? Couldn’t it be entirely plausible that these areas are places with low speeds as well? (Which is what I was thinking in real time) Thanks.
 Adam Tyson
PowerScore Staff
  • PowerScore Staff
  • Posts: 5153
  • Joined: Apr 14, 2011
|
#48127
Thanks for the question, ltowns1! The "other" in that answer choice refers to "other than areas with lower speed limits", which were the only areas about which the author gave evidence. The author said that fatalities are on the rise in areas with lower speed limits, and concludes that lower speed limits must be bad and higher ones would be better. It seems that he assumed that the rate is rising only in those areas. But what if they are also rising in other areas? That is, areas other than those with lower speed limits? What if they are rising everywhere? Then his argument falls apart, because there wouldn't be any evidence that the lower speed limits are the problem (and in fact, they might be helping).

I can see how you might confuse this with a claim about "other than in our geographic area", but that's not how the author framed his argument. His evidence wasn't about any particular geographic area, but ALL areas with lower speed limits.

Does that shed a little more light on the matter? If not, let us know!
 ltowns1
  • Posts: 61
  • Joined: May 16, 2017
|
#48631
Yes it does! Thanks Adam!
User avatar
 Albertlyu
  • Posts: 98
  • Joined: Jul 18, 2020
|
#84239
Adam Tyson wrote: Thu Jul 19, 2018 5:00 pm Thanks for the question, ltowns1! The "other" in that answer choice refers to "other than areas with lower speed limits", which were the only areas about which the author gave evidence. The author said that fatalities are on the rise in areas with lower speed limits, and concludes that lower speed limits must be bad and higher ones would be better. It seems that he assumed that the rate is rising only in those areas. But what if they are also rising in other areas? That is, areas other than those with lower speed limits? What if they are rising everywhere? Then his argument falls apart, because there wouldn't be any evidence that the lower speed limits are the problem (and in fact, they might be helping).

I can see how you might confuse this with a claim about "other than in our geographic area", but that's not how the author framed his argument. His evidence wasn't about any particular geographic area, but ALL areas with lower speed limits.

Does that shed a little more light on the matter? If not, let us know!
thank you, Adam. Got this one wrong as I did not find the right conclusion, "that won't be true for long" or "the evidence actually supports the view....", I thought it was the second, hence the flaw would be the author offered no evidence to back his conclusion, just because the fatality rates in every area are rising does not mean the speed limits should be increased. I now see what is wrong with E and why D is right even if the main conclusion is the second one. Please may I ask if the second one is the main conclusion? thanks。

Albert
 Katherinthesky
  • Posts: 36
  • Joined: Feb 07, 2020
|
#86289
Hello,

While I'm not a fan of (D) due to its vagueness, I can understand why it's correct.

However, I can't shake off (A) because the letter writer mentions the evidence in the article as "distorted" - then bases his own conclusion (about how speed limits should be increased) on that same evidence that he considers distorted.

What am I misunderstanding here?

Any help greatly appreciated, thanks in advance!
 Robert Carroll
PowerScore Staff
  • PowerScore Staff
  • Posts: 1787
  • Joined: Dec 06, 2013
|
#86916
Katherin,

I think I have two objections to answer choice (A):

1. It's not actually clear that the "evidence" in the conclusion is from the study. The sentence before the conclusion is where the author gets his/her evidence from, and it doesn't say that's definitely from the study.

2. The author's objection is to the "distorted evidence" in the study. If the author uses evidence from the study, the author could well say that they're not distorting it. The author isn't really committed to the idea that the study is entirely worthless, but that the "flawed reasoning" and "distorted evidence" are problematic. The author could simply say "Well, use the evidence without distorting it, and when you reason from that UNdistorted evidence, make certain to commit no flaws." It doesn't look like the author thinks no proper use can be made of the study at all.

I think 2 is more clearly a reason why answer choice (A) is out than 1.

Robert Carroll
User avatar
 ashpine17
  • Posts: 321
  • Joined: Apr 06, 2021
|
#98252
I conflated lower for low…so if the argument had said low, it would have been a good argument?

Get the most out of your LSAT Prep Plus subscription.

Analyze and track your performance with our Testing and Analytics Package.